0% found this document useful (0 votes)
53 views4 pages

Midterm Solutions: N N N N N N N N N N N

This document contains solutions to 8 problems related to analysis: 1) It proves the convergence of (|xn|) given convergence of (xn) and gives a counterexample. 2) It shows an unbounded, non-convergent sequence where the terms grow exponentially. 3) It proves a sequence is Cauchy if the differences between terms decrease exponentially. 4) It uses equicontinuity and boundedness to show a sequence of functions has a convergent subsequence. 5) It proves the limit of integrals is equal to the integral of the limit of functions. 6) It gives a counterexample showing a function cannot be uniformly approximated by polynomials. 7

Uploaded by

Nitish Kumar
Copyright
© © All Rights Reserved
We take content rights seriously. If you suspect this is your content, claim it here.
Available Formats
Download as PDF, TXT or read online on Scribd
0% found this document useful (0 votes)
53 views4 pages

Midterm Solutions: N N N N N N N N N N N

This document contains solutions to 8 problems related to analysis: 1) It proves the convergence of (|xn|) given convergence of (xn) and gives a counterexample. 2) It shows an unbounded, non-convergent sequence where the terms grow exponentially. 3) It proves a sequence is Cauchy if the differences between terms decrease exponentially. 4) It uses equicontinuity and boundedness to show a sequence of functions has a convergent subsequence. 5) It proves the limit of integrals is equal to the integral of the limit of functions. 6) It gives a counterexample showing a function cannot be uniformly approximated by polynomials. 7

Uploaded by

Nitish Kumar
Copyright
© © All Rights Reserved
We take content rights seriously. If you suspect this is your content, claim it here.
Available Formats
Download as PDF, TXT or read online on Scribd
You are on page 1/ 4

Midterm Solutions

1. Prove that if (xn ) of real numbers is convergent then (|xn |) is also


convergent. Give an example to show that the convergence of (|xn |)
need not imply the convergence of (xn ).
Solution. Since (xn ) is convergent, given  > 0, there exists N N
such that |xn x| < , for all n N .
From the triangle inequality, we know that ||xn | |x|| |xn x|.
Consequently, we have that ||xn | |x|| < , for all n N , which proves
the convergence of (|xn |).
The sequence ((1)n ) is non-convergent, but converges absolutely to 1.
2. Let X = (xn ) be a sequence of positive real numbers such that
 
xn+1
lim = L > 1.
xn
Show that X is not a bounded sequence and hence is not convergent.
 
Solution. Since lim xxn+1 n
= L, given  > 0, there exists N N such

that xxn+1 L < , for all n N . This would imply that for n N ,

n
xn+1
xn
> L  = r, where r > 1 for sufficiently small .
Therefore, for n N , we obtain
xn+1 > rxn > r2 xn1 > . . . > xN rnN +1 .
By setting C = xN /rN , we get xn+1 > Crn+1 for all n N . Since
r > 1, this sequence will be unbounded, and hence diverges.
3. If 0 < r < 1 and |xn+1 xn | < rn for all n N, show that (xn ) is a
Cauchy sequence.
Solution. For n > m, we have that
n n
X X
i1 rm rn
|xn xm | |xi xi1 | < r = .
i=m+1 i=m+1
1r

Since 0 < r < 1, lim(rn ) = 0, and hence (rn ) is Cauchy. Therefore,


given  > 0, there exists N N such that for all m, n N , we have
|rm rn | < (1 r).

1
Hence for all m, n N , we have that |xn xm | < , showing that (xn )
is Cauchy.

4. Let (fn ) C[0, 1] be such that there exists M > 0 such that kfn k
M , for all n N. Define
Z x
Fn (x) = fn (t) dt.
0

Show that Fn has a uniformly convergent subsequence.


Solution. From the Fundamental Theorem of Calculus, we have that
Fn0 (x) = fn (x), and hence kFn0 k = kfn k M .
By applying the Mean Value Theorem to the interval [x, y] [0, 1], we
obtain a c (x, y) such that Fn (y) Fn (x) = Fn0 (c)(y x). This would
imply that

|Fn (y) Fn (x)| kFn0 k |y x| = M |y x|. (1)

Therefore, given  > 0, if we choose = /M , we have that |Fn (y)


Fn (x)| < , whenever |xy| < . This shows that the set {Fn : n N}
is equicontinuous.
Moreover, putting x = 0 in Equation (1), we obtain |Fn (y)| M |y|
M , and taking supremum over all y [0, 1], we get that kFn k M ,
that is, Fn is bounded for all n N.
From the Corollary to the Ascoli-Arzela Theorem, we conclude that
(Fn ) has a uniformly convergent subsequence.
R1 R1
5. Let lim(fn ) = f in C[0, 1]. Then show that lim 0 fn (x) dx = 0 f (x) dx.
Solution. Since lim(fn ) = f , given  > 0 and x [0, 1], there exists
N = N (x, ) N such that |fn (x) f (x)| < , for all n N .
Therefore, for all n N , we have that
Z 1 Z 1

(fn (x) f (x)) dx |fn (x) f (x)| dx <  .

0 0

R1 R1
This shows that lim 0
fn (x) dx = 0
f (x) dx.

2
6. Let f : (0, 1) R given by f (x) = 1/x cannot be approximated
uniformly by polynomials on (0, 1). In other words, show that given
 > 0 there exists no polynomial p(x) on (0, 1) such that kp f k < .
Solution. Suppose that given  > 0, there exists a polynomial p such
that kf pk < . Then this would imply that kf k < kpk + ,
showing that f is bounded in (0, 1). But this is a contradiction to the
fact that 1/x is unbouded (0, 1).
7. Consider the metric space

X
`2 = {(xn ) : xi R and |xn |2 < }
n=1

with the usual metric. Consider the sequence (en ) `2 given by


en = (0, 0, . . . , 0, 1, 0, 0, . . .)
(ie. en has 1 in the nth position and zero elsewhere). Show that (en ) is
a bounded sequence, but does not have a convergent subsequence.
Solution. Let (an ) be a sequence of sequences in `2 . We will denote
the ith term of the nth sequence an by an (i). For the sequence to be
bounded, we need to have a M > 0 such that kxn k2 < M , that is,
P 2
i=1 |an (i)| < , for all n N. Since ken k2 = 1 for all n, we have
that the sequence (en ) is bounded.
Suppose that (ank ) is a convergent
subesequence of (an ). Then we
can see that kank anm k2 = 2, for all k, m N, which shows that
the subsequence will not be Cauchy. Hence no subsequence can be
convergent.
8. (Bonus) Consider the space C[0, 1], and define
Z 1
kf k1 = |f (t)| dt
0

Show that k k1 is a norm on C[0, 1].


Solution.
(i) Since |(f (t)| R0, we have that kf k1 0. Moreover, since f is
1
continuous and 0 |f (t)| dt = 0, which would imply that f (t) = 0,
for all t [0, 1]. (This is an exercise in calculus.)

3
(ii) For c R and f C[0, 1],
Z 1 Z 1
kcf k1 = |cf (t)| dt = |c| |f (t)| dt = |c|kf k1 .
0 0

(iii) For f, g C[0, 1], we have that


Z 1 Z 1 Z 1
kf +gk1 = |f (t)+g(t)| dt |f (t)| dt+ |g(t)| dt = kf k1 +kgk1 .
0 0 0

You might also like